in Calculus retagged by
2,375 views
10 votes
10 votes

$$\int_{0}^{1} \log_e(x) dx=$$

  1. $1$
  2. $-1$
  3. $\infty $
  4. $-\infty $
  5. None of the above
in Calculus retagged by
2.4k views

3 Comments

1
1
try to solve using graph three option(a,c,d) will be eliminated instantly...
0
0
edited by

Can be solved by putting Taylor expansion of natural logarithm, that is:

$\ln(x) = \left(x-1\right)-\frac{1}{2}\left(x-1\right)^2 + \frac{1}{3} \left(x-1\right)^3-\frac{1}{4} \left(x-1\right)^4 + \cdots$

More terms we add, closer the result approaches to -1.

0
0

3 Answers

9 votes
9 votes
Best answer

Use Integration by Parts

$\large\int \ln(x) dx$

set 
  $u = \ln(x),$    $dv = dx$ 
then we find 
  $du = \left(\frac{1}{x}\right) dx,$    $v = x$

substitute

$\large\int \ln(x) dx =\large\int u\; dv$

and use integration by parts

$= uv - \large\int v \;du$

substitute $u=\ln(x), v=x,$ and $du=\left(\frac{1}{x}\right)dx$

$= \ln(x)\; {x}- \large\int x \left(\frac{1}{x}\right) dx$ 
$= \ln(x) x -\large\int dx$ 
$= \ln(x) x - x + C$ 
$= x \ln(x) - x + C.$ 

Now Put Limits

$[\ln(1)-1+C]-[0-0+C]= -1$

Note-

$\lim [x\ln x] = 0.$
 $x=0$

Correct Answer: $B$

edited by
by

4 Comments

@sid, answer is -1

You can  confirm here.

1
1
Given question is an example of an Improper integral of $2^{nd}$ kind.

here ,  at $x = 0 $ , $lnx$ is undefined . So, given integral may or may not be existed. We have to check whether it converges or not.

According to definition of Improper integral of $2^{nd}$ kind :-

$\int_{0}^{1} f(x) \;dx= \lim_{a\rightarrow 0^{+}}\int_{a}^{1} f(x) \;dx$

So, here ,

$\Rightarrow$$\int_{0}^{1} lnx \;dx= \lim_{a\rightarrow 0^{+}}\int_{a}^{1} 1*lnx \;dx$

=$\lim_{a\rightarrow 0^{+}} (xlnx -x) | _{a}^{1}$

= $0 - 1 - \lim_{a\rightarrow 0^{+}} alna - 0$

= -1 - $\lim_{a\rightarrow 0^{+}} \frac{lna}{\frac{1}{a}}$

= -1 - $\lim_{a\rightarrow 0^{+}} \frac{\frac{1}{a}}{\frac{-1}{a^{2}}}$ (Using L'H$\hat{o}$pital's Rule)

= -1 - 0

= -1

It means given integral converges and gives finite area under the curve within the given limits and its value evaluated as -1
12
12
edited by

@ankitgupta.1729 

Perfect !!

There is a flaw in solution as best answer. Admin should change it.

1
1
6 votes
6 votes
$\int ln(x) dx$

= $\int ln(x) * 1 dx$

= $ln(x) \int 1dx - \int (1/x\int 1dx)dx$

= $ln(x)x- \int 1 dx$

= $ln(x) * x - x(1-0)$ + C

= $xln(x) - x + C$

 

Applying limits

$1*ln(1) -1 + C - (0 - 0 +C)$

$= -1$ is the answer.
3 votes
3 votes

I try to do some other way

edited by

4 Comments

edited by
How you calculate  the lower limit and upper limit after changing  it from dx to dt.as ln0 is undefined how to calculate  the limit for it.
0
0
edited by
$0\times \infty=0$ right?

No this is actually indeterminate form.
0
0
1
1
Okay, you are right.

I update the solution.
0
0
Answer:

Related questions

Quick search syntax
tags tag:apple
author user:martin
title title:apple
content content:apple
exclude -tag:apple
force match +apple
views views:100
score score:10
answers answers:2
is accepted isaccepted:true
is closed isclosed:true